Biographies Characteristics Analysis

The system of experimental homework in physics using children's toys. Experiment in physics

Home experimental tasks

Exercise 1.

Take a long heavy book, tie it with a thin thread and

attach a rubber thread 20 cm long to the thread.

Put the book on the table and very slowly begin to pull on the end.

rubber thread. Try to measure the length of the stretched rubber thread in

the moment the book starts sliding.

Measure the length of the stretched thread with the book moving evenly.

Place two thin cylindrical pens under the book (or two

cylindrical pencil) and also pull the end of the thread. Measure length

stretched thread with uniform movement of the book on the rollers.

Compare the three results and draw conclusions.

Note. The next task is a variation of the previous one. It

also aimed at comparing static friction, sliding friction and friction

Task 2.

Place a hexagonal pencil on top of the book parallel to the spine.

Slowly lift the top edge of the book until the pencil starts

slide down. Slightly reduce the slope of the book and secure it in this

position by putting something under it. Now the pencil if its over

put on the book, will not move out. It is held in place by the force of friction.

static friction force. But it is worth weakening this force a little - and for this it is enough

flick your finger on the book - and the pencil will crawl down until it falls on

table. (The same experiment can be done, for example, with a pencil case, match

box, eraser, etc.)

Think about why it is easier to pull a nail out of the board if you rotate it

around the axis?

To move a thick book on the table with one finger, you need to attach

some effort. And if you put two round pencils under the book or

pens that will be in this case roller bearings, book easily

will move from a weak push with the little finger.

Do experiments and make a comparison of the static friction force, the friction force

sliding and rolling friction forces.

Task 3.

In this experiment, two phenomena can be observed at once: inertia, experiments with

Take two eggs, one raw and one hard boiled. spin

both eggs on a large plate. You see that the boiled egg behaves differently,

than raw: it rotates much faster.

In a boiled egg, the white and yolk are rigidly bonded to their shell and

among themselves because are in a solid state. And when we spin

raw egg, then at first we unwind only the shell, only then, due to

friction, layer by layer, rotation is transferred to the protein and yolk. Thus,

liquid protein and yolk, by their friction between the layers, slow down the rotation

shells.

Note. Instead of raw and boiled eggs, you can spin two pans,

in one of which there is water, and in the other there is the same amount of cereals by volume.

Center of gravity. Exercise 1.

Take two faceted pencils and hold them in front of you parallel,

putting a line on them. Start bringing the pencils closer together. Rapprochement will

occur in alternating movements: then one pencil moves, then the other.

Even if you want to interfere with their movement, you will not succeed.

They will still move forward.

As soon as on one pencil the pressure became greater and the friction

the second pencil can now move under the ruler. But after some

time, the pressure over it becomes greater than over the first pencil, and

as friction increases, it stops. And now the first one can move

pencil. So, moving in turn, the pencils will meet in the very middle

ruler at its center of gravity. This can be easily verified by the divisions of the ruler.

This experiment can also be done with a stick, holding it on outstretched fingers.

As you move your fingers, you will notice that they, also moving alternately, will meet

under the very middle of the stick. True, this is only special case. Try

do the same with a regular broom, shovel or rake. You

you will see that the fingers will not meet in the middle of the stick. Try to explain

why is this happening.

Task 2.

This is very old visual experience. Penknife (folding) you have,

probably a pencil too. Sharpen your pencil so it has a sharp end

and stick a half-open penknife a little above the end. Put

the point of a pencil on the index finger. Find such a position

half-open knife on a pencil, in which the pencil will stand on

finger, slightly swaying.

Now the question is: where is the center of gravity of the pencil and pen

Task 3.

Determine the position of the center of gravity of a match with and without a head.

Place a matchbox on the table on its long narrow edge and

put a match without a head on the box. This match will serve as a support for

another match. Take a match with a head and balance it on a support so that

so that it lies horizontally. Mark the position of the center of gravity with a pen

matches with a head.

Scrape the head off the match and place the match on a support so that

the ink dot you marked was on the support. It's not for you now

succeed: the match will not lie horizontally, since the center of gravity of the match

moved. Determine the position of the new center of gravity and notice in

which side he moved. Mark with a pen the center of gravity of the match without

Bring a match with two dots to class.

Task 4.

Determine the position of the center of gravity of a flat figure.

Cut out a figure of arbitrary (any bizarre) shape from cardboard

and pierce several holes in different arbitrary places (better if

they will be located closer to the edges of the figure, this will increase the accuracy). Drive in

into a vertical wall or rack a small carnation without a cap or a needle and

hang a figure on it through any hole. Notice the shape

should swing freely on the stud.

Take a plumb line, consisting of a thin thread and weight, and throw it over

thread through the stud so that it indicates the vertical direction is not

suspended figure. Mark the vertical direction on the figure with a pencil

Remove the figure, hang it in any other hole and again with

Using a plumb line and a pencil, mark the vertical direction of the thread on it.

The intersection point of the vertical lines will indicate the position of the center of gravity

this figure.

Pass a thread through the center of gravity you found, at the end of which

a knot is made, and hang the figure on this thread. The figure must be kept

almost horizontal. The more accurately the experience is done, the more horizontal it will be.

keep figure.

Task 5.

Determine the center of gravity of the hoop.

Take a small hoop (for example, a hoop) or make a ring out of

flexible twig, from a narrow strip of plywood or hard cardboard. hang up

it on a stud and lower the plumb line from the hanging point. When the plumb line

calm down, mark on the hoop the points of her touch to the hoop and between

stretch and fasten a piece of thin wire or fishing line with these points

(you need to pull hard enough, but not so much that the hoop changes its

Hang the hoop on a stud at any other point and do the same

most. The intersection point of the wires or lines will be the center of gravity of the hoop.

Note: the center of gravity of the hoop lies outside the substance of the body.

Tie a thread to the intersection of wires or lines and hang it on

her hoop. The hoop will be in indifferent equilibrium, since the center

the gravity of the hoop and the point of its support (suspension) coincide.

Task 6.

You know that the stability of the body depends on the position of the center of gravity and

on the size of the support area: the lower the center of gravity and the larger the support area,

the more stable the body.

With this in mind, take a bar or an empty matchbox and, placing it

alternately on paper in a box to the widest, to the middle and to the most

smaller side, circle each time with a pencil to get three different

support area. Calculate the size of each area in square centimeters

and write them down on paper.

Measure and record the height of the center of gravity of the box for all

three cases (center of gravity matchbox lies at the intersection

diagonals). Conclude at what position of the boxes is the most

sustainable.

Task 7.

Sit on a chair. Place your feet vertically without slipping them under

seat. Sit completely straight. Try to stand up without bending forward

without stretching your arms forward and without moving your legs under the seat. you have nothing

succeed - you won't be able to get up. Your center of gravity, which is located somewhere

in the middle of your body, will not let you get up.

What condition must be met in order to get up? Gotta lean forward

or tuck your feet under the seat. When we get up, we always do both.

In this case, the vertical line passing through your center of gravity should

be sure to go through at least one of the soles of your feet or between them.

Then the balance of your body will be stable enough, you can easily

you can get up.

Well, now try to stand up, picking up dumbbells or an iron. Pull out

hands forward. You may be able to stand up without bending over or bending your legs under

Inertia. Exercise 1.

Put a postcard on the glass, and put a coin on the postcard

or checker so that the coin is above the glass. Hit the postcard

click. The postcard should fly out, and the coin (checker) should fall into the glass.

Task 2.

Place a double sheet of notebook paper on the table. For one half

sheet, put a stack of books at least 25 cm high.

Slightly lifting the second half of the sheet above the level of the table with both

hands, quickly pull the sheet towards you. The sheet should come out from under

books, and the books should stay where they are.

Put the book back on the sheet and pull it now very slowly. Books

will move with the sheet.

Task 3.

Take a hammer, tie a thin thread to it, but so that it

withstood the weight of a hammer. If one thread fails, take two

threads. Slowly lift the hammer up by the thread. The hammer will hang on

thread. And if you want to raise it again, but not slowly, but quickly

jerk, the thread will break (ensure that the hammer, falling, does not break

nothing underneath). The inertia of the hammer is so great that the thread does not

survived. The hammer did not have time to quickly follow your hand, remained in place, and the thread broke.

Task 4.

Take a small ball made of wood, plastic or glass. Make out

thick paper groove, put a ball in it. Move quickly across the table

groove, and then suddenly stop it. The inertia ball will continue

movement and roll, jumping out of the groove.

Check where the ball will roll if:

a) pull the chute very quickly and stop it abruptly;

b) pull the chute slowly and stop abruptly.

Task 5.

Cut the apple in half, but not all the way through, and let it hang

Now hit with the blunt side of the knife with the apple hanging on top of it on

something hard, like a hammer. Apple, moving on

inertia, will be cut and split into two halves.

The same thing happens when wood is chopped: if it was not possible

split a block of wood, they usually turn it over and, with all their strength, hit it with a butt

an ax on a solid support. Churbak, continuing to move by inertia,

is planted deeper on the ax and splits in two.

The meaning and types of independent experiment of students in physics. When teaching physics in high school, experimental skills are formed when performing independent laboratory work.

Teaching physics cannot be presented only in the form of theoretical classes, even if students are shown demonstration physical experiments in the classroom. To all types of sensory perception, it is necessary to add “work with hands” in the classroom. This is achieved when students perform a laboratory physical experiment, when they themselves assemble installations, measure physical quantities, and perform experiments. Laboratory classes arouse great interest among students, which is quite natural, since in this case the student learns the world around him on the basis of own experience and your own feelings.

The significance of laboratory classes in physics lies in the fact that students form ideas about the role and place of the experiment in cognition. When performing experiments, students develop experimental skills, which include both intellectual and practical skills. The first group includes skills: to determine the purpose of the experiment, to put forward hypotheses, to select instruments, to plan an experiment, to calculate errors, to analyze results, to draw up a report on the work done. The second group includes skills: to assemble an experimental setup, to observe, measure, experiment.

In addition, the significance of a laboratory experiment lies in the fact that when it is performed, students develop such important personal qualities, as accuracy in the work of devices; observance of cleanliness and order in the workplace, in the records that are made during the experiment, organization, perseverance in obtaining results. They form a certain culture of mental and physical labor.

In the practice of teaching physics at school, three types of laboratory classes have developed:

Frontal laboratory work in physics;

Physical workshop;

Home experimental work in physics.

Frontal laboratory work- this is a type of practical work when all students in the class simultaneously perform the same type of experiment using the same equipment. Frontal laboratory work is most often carried out by a group of students consisting of two people, sometimes it is possible to organize individual work. Accordingly, the office should have 15-20 sets of instruments for frontal laboratory work. The total number of such devices will be about a thousand pieces. The names of frontal laboratory works are given in curricula. There are a lot of them, they are provided for almost every topic of the physics course. Before carrying out the work, the teacher reveals the preparedness of the students for the conscious performance of the work, determines with them its purpose, discusses the progress of the work, the rules for working with instruments, methods for calculating measurement errors. Frontal laboratory work is not very complex in content, is closely related chronologically to the material being studied and is usually designed for one lesson. Descriptions of laboratory work can be found in school textbooks in physics.

Physical workshop is carried out with the aim of repeating, deepening, expanding and generalizing the knowledge gained from different topics physics course; development and improvement of students' experimental skills through the use of more sophisticated equipment, a more complex experiment; the formation of their independence in solving problems related to the experiment. The physical workshop is not connected in time with the material being studied, it is usually held at the end of the academic year, sometimes at the end of the first and second semesters and includes a series of experiments on a particular topic. Students perform the work of a physical workshop in a group of 2-4 people using various equipment; in the following classes there is a change of work, which is done according to a specially drawn up schedule. When scheduling, take into account the number of students in the class, the number of workshops, the availability of equipment. For each work of the physical workshop, two teaching hours, which requires the introduction of dual lessons in physics into the schedule. This presents difficulties. For this reason, and due to the lack of necessary equipment, one-hour work of a physical workshop is practiced. It should be noted that two-hour work is preferable, since the work of the workshop is more difficult than frontal laboratory work, they are performed on more sophisticated equipment, and the proportion of students' independent participation is much larger than in the case of frontal laboratory work. Physical practicums are provided basically by programs of 9-11 classes. Approximately 10 hours of study time is allotted for each class. For each work, the teacher must draw up an instruction that should contain: name, purpose, list of instruments and equipment, a brief theory, a description of instruments unknown to students, a work plan. After completing the work, students must submit a report that should contain: the name of the work, the purpose of the work, a list of instruments, a diagram or drawing of the installation, a work execution plan, a table of results, formulas by which the values ​​\u200b\u200bof were calculated, calculation of measurement errors, conclusions. When evaluating the work of students in the workshop, one should take into account their preparation for work, a report on the work, the level of skills development, understanding of the theoretical material, the methods of experimental research used.

Home experimental work. Home laboratory work is the simplest independent experiment that is performed by students at home, outside of school, without direct control from the teacher over the progress of work.

The main tasks of this type of experimental work are:

Formation of the ability to observe physical phenomena in nature and in everyday life;

Formation of the ability to perform measurements with the help of measuring instruments used in everyday life;

Formation of interest in experiment and in the study of physics;

Formation of independence and activity.

Home laboratory work can be classified depending on the equipment used in their performance:

Works that use household items and improvised materials (measuring cup, tape measure, household scales, etc.);

Works in which home-made devices are used (lever scales, electroscope, etc.);

Work performed on industrial devices.

The classification is taken from .

In his book S.F. Pokrovsky showed that home experiments and observations in physics carried out by the students themselves: 1) make it possible for our school to expand the area of ​​connection between theory and practice; 2) develop students' interest in physics and technology; 3) awaken creative thought and develop the ability to invent; 4) accustom students to independent research work; 5) develop valuable qualities in them: observation, attention, perseverance and accuracy; 6) supplement classroom laboratory work with material that cannot be done in class in any way (a series of long-term observations, observation natural phenomena and so on), and 7) accustom students to conscious, expedient work.

Home experiments and observations in physics have their own characteristics, being an extremely useful addition to class and general school practical work.

It has long been recommended that students have a home laboratory. it included, first of all, rulers, a beaker, a funnel, scales, weights, a dynamometer, a tribometer, a magnet, a watch with a second hand, iron filings, tubes, wires, a battery, a light bulb. However, despite the fact that very simple instruments are included in the set, this proposal has not been adopted.

To organize the home experimental work of students, you can use the so-called mini-laboratory proposed by the teacher-methodologist E.S. Obedkov, which includes many household items (penicillin bottles, rubber bands, pipettes, rulers, etc.), which is available to almost every student. E.S. Obyedkov developed a very big number interesting and useful experiences with this equipment.

It also became possible to use a computer to conduct a model experiment at home. It is clear that the corresponding tasks can only be offered to those students who have a computer and software and pedagogical tools at home.

For students to want to learn, it is necessary that the learning process is interesting for them. What are the students interested in? To get an answer to this question, we turn to excerpts from the article by I.V. Litovko, MOS (P) Sh No. 1 of Svobodny “Home experimental tasks as an element of students' creativity”, published on the Internet. Here is what I.V. Litovko:

“One of the most important tasks of the school is to teach students how to learn, to strengthen their ability for self-development in the process of education, for which it is necessary to form appropriate stable desires, interests, and skills in schoolchildren. An important role in this is played by experimental tasks in physics, which in their content represent short-term observations, measurements and experiments that are closely related to the topic of the lesson. The more observations of physical phenomena, experiments the student makes, the better he will master the material being studied.

To study the motivation of students, they were asked the following questions and the results were obtained:

What do you like about studying physics ?

a) problem solving -19%;

b) demonstration of experiments -21%;


Introduction

Chapter 1

1 The role and significance of experimental tasks in the school course of physics (the definition of an experiment in pedagogy, psychology and in the theory of methods of teaching physics)

2 Analysis of programs and textbooks on the use of experimental tasks in the school physics course

3 A new approach to conducting experimental tasks in physics using Lego-constructors on the example of the section "Mechanics"

4 Methodology for conducting a pedagogical experiment at the level of a stating experiment

5 Conclusions on the first chapter

Chapter 2

1 Development of systems of experimental tasks on the topic "Kinematics of a point". Methodological recommendations for use in physics lessons

2 Development of systems of experimental tasks on the topic "Rigid Body Kinematics". Methodological recommendations for use in physics lessons

3 Development of systems of experimental tasks on the topic "Dynamics". Methodological recommendations for use in physics lessons

4 Development of systems of experimental tasks on the topic "Conservation laws in mechanics". Methodological recommendations for use in physics lessons

5 Development of systems of experimental tasks on the topic "Statics". Methodological recommendations for use in physics lessons

6 Conclusions on the second chapter

Conclusion

Bibliography

Answer to the question


Introduction


Relevance of the topic. It is generally recognized that the study of physics provides not only factual knowledge, but also develops the personality. Physical education is undoubtedly the sphere of the development of the intellect. The latter, as is known, manifests itself both in the mental and in the objective activity of a person.

In this regard, of particular importance is the experimental solution of problems, which necessarily involves both types of activity. Like any kind of problem solving, it has a structure and patterns common to the process of thinking. The experimental approach opens up opportunities for the development of figurative thinking.

The experimental solution of physical problems, due to their content and solution methodology, can become an important means of developing universal research skills and abilities: setting up an experiment based on certain research models, experimenting itself, the ability to identify and formulate the most significant results, put forward a hypothesis adequate to the subject being studied , and on its basis to build a physical and mathematical model, to involve computer technology in the analysis. The novelty of the content of physical problems for students, the variability in the choice of experimental methods and means, the necessary independence of thinking in the development and analysis of physical and mathematical models create prerequisites for the formation of creative abilities.

Thus, the development of a system of experimental tasks in physics using the example of mechanics is relevant in terms of developmental and student-centered education.

The object of the study is the process of teaching tenth grade students.

The subject of the study is a system of experimental tasks in physics on the example of mechanics, aimed at developing intellectual abilities, formation research approach, creative activity students.

The purpose of the study is to develop a system of experimental tasks in physics using the example of mechanics.

Research hypothesis - If the system of the physical experiment of the "Mechanics" section includes teacher demonstrations, related home and classroom experiments of students, as well as experimental tasks for students in elective courses, and cognitive activity students during their implementation and discussion to organize on the basis of problems, then schoolchildren will have the opportunity to acquire, along with knowledge of the basic physical concepts and laws, information, experimental, problem, activity skills, which will lead to an increase in interest in physics as a subject. Based on the purpose and hypothesis of the study, the following tasks were delivered:

1. Determine the role and significance of experimental tasks in the school physics course (the definition of an experiment in pedagogy, psychology and in the theory of methods of teaching physics).

To analyze programs and textbooks on the use of experimental tasks in the school physics course.

To reveal the essence of the methodology for conducting a pedagogical experiment at the level of a stating experiment.

To develop a system of experimental tasks in the section "Mechanics" for students in grade 10 of a general education profile.

The scientific novelty and theoretical significance of the work is as follows: The role of the experimental solution of physical tasks as a means in the development of cognitive abilities, research skills and creative activity of 10th grade students is established.

The theoretical significance of research is determined by the development and justification methodological foundations technologies for designing and organizing the educational process for the experimental solution of physical problems as a means of developing and student-centered learning.

To solve the tasks set, a set of methods was used:

· theoretical analysis of psychological and pedagogical literature and comparative methods;

· systems approach to the evaluation of the results of theoretical analysis, the method of ascent from the abstract to the concrete, the synthesis of theoretical and empirical material, the method of meaningful generalization, the logical-heuristic development of solutions, probabilistic forecasting, predictive modeling, thought experiment.

The work consists of an introduction, two chapters, conclusion, bibliography, applications.

Approbation of the developed system of tasks was carried out on the basis of boarding school No. 30 of the Secondary General Education of the Open Joint Stock Company "Russian Railways", address: Komsomolsk - on the Amur, Lenin Avenue 58/2.


Chapter 1


1 The role and significance of experimental tasks in the school course of physics (the definition of an experiment in pedagogy, psychology and in the theory of methods of teaching physics)


Robert Woodworth, who published his classic textbook on experimental psychology("Experimental psychology", 1938), defined an experiment as an ordered study in which the researcher directly changes a certain factor (or factors), keeps the others unchanged, and observes the results of systematic changes.

In pedagogy, V. Slastenin defined an experiment as a research activity with the aim of studying cause-and-effect relationships in pedagogical phenomena.

In philosophy Sokolov V.V. describes the experiment as a method of scientific knowledge.

The founder of physics - Znamensky A.P. described the experiment as a kind of cognitive activity in which the key situation for a particular scientific theory is played out not in real action.

According to Robert Woodworth, a stating experiment is an experiment that establishes the existence of some immutable fact or phenomenon.

According to V. Slastenin - a stating experiment is carried out at the beginning of the study and is aimed at clarifying the state of affairs in school practice on the problem under study.

According to Robert Woodworth, a formative (transforming, teaching) experiment aims to actively form or educate certain aspects of the psyche, levels of activity, etc.; is used in the study of specific ways of forming the child's personality, providing a connection psychological research with pedagogical search and design of the most effective forms educational work.

According to Slastenin, V. is a formative experiment, during which new pedagogical phenomena are constructed.

According to V. Slastenin - experimental tasks are short-term observations, measurements and experiments that are closely related to the topic of the lesson.

personally oriented learning- this is such training, where the personality of the child, its originality, self-worth, is at the forefront, the subjective experience of each is first revealed, and then coordinated with the content of education. If in the traditional philosophy of education socio-pedagogical models of personality development were described in the form of externally set samples, standards of cognition (cognitive activity), then personality-oriented learning proceeds from the recognition of the uniqueness of the subjective experience of the student himself, as an important source of individual life activity, manifested, in particular, in cognition. Thus, it is recognized that in education there is not just an internalization by the child of given pedagogical influences, but a “meeting” of the given and subjective experience, a kind of “cultivation” of the latter, its enrichment, increment, transformation, which constitutes the “vector” individual development The recognition of the student as the main acting figure in the entire educational process is personality-oriented pedagogy.

When designing the educational process, one must proceed from the recognition of two equal sources: teaching and learning. The latter is not just a derivative of the former, but is an independent, personally significant, and therefore a very effective source of personality development.

Student-centered learning is based on the principle of subjectivity. It follows from it whole line provisions.

Learning material cannot be the same for all students. The student should be given the opportunity to choose what corresponds to his subjectivity when studying the material, completing tasks, solving problems. In the content of educational texts, contradictory judgments, variability of presentation, manifestation of different emotional attitude, copyright positions. The student does not memorize the required material with predetermined conclusions, but selects it himself, studies, analyzes and draws his own conclusions. The emphasis is not only on the development of the student's memory, but on the independence of his thinking and the originality of his conclusions. The problematic nature of tasks, the ambiguity of the educational material push the student to this.

A formative experiment is a type of experiment that is specific exclusively to psychology, in which the active influence of the experimental situation on the subject should contribute to his mental development and personal growth.

Let us consider the role and significance of experimental tasks in psychology, pedagogy, philosophy, and the theory of methods of teaching physics.

main method research work psychologist is an experiment. Well-known domestic psychologist S.L. Rubinstein (1889-1960) singled out the following qualities of the experiment, which determine its importance for obtaining scientific facts: “1) In the experiment, the researcher himself causes the phenomenon he is studying, instead of waiting, as in objective observation, until the random flow of the phenomenon gives him the opportunity to observe it . 2) Having the opportunity to cause the phenomenon under study, the experimenter can vary, change the conditions under which the phenomenon occurs, instead of, as in simple observation, taking them as they are given to him by chance. 3) By isomering individual conditions and changing one of them while keeping the rest unchanged, the experiment thereby reveals the significance of these individual conditions and establishes regular connections that determine the process being studied. Experiment is thus a very powerful methodological tool for identifying patterns. 4) By revealing regular connections between phenomena, an experiment can often vary not only the conditions themselves in the sense of their presence or absence, but also their quantitative ratios. As a result, the experiment establishes qualitative patterns that allow mathematical formulation.

The brightest pedagogical direction Experimental pedagogy, the leading aspiration of which is the development of a scientifically based theory of education and upbringing, capable of developing the individuality of the individual, is called upon to realize the ideas of the “new education”. Emerged in the 19th century experimental pedagogy (the term was proposed by E. Meiman) aimed at a comprehensive study of the child and experimental substantiation of pedagogical theory. She rendered strong influence on the course of development of domestic pedagogical science. .

No topic should be dealt with purely theoretically, just as no work should be done without elucidating its scientific theory. A skillful combination of theory with practice and practice with theory will give the necessary educational and educational effect and ensure the fulfillment of the requirements that pedagogy imposes on us. The main tool for teaching physics (its practical part) at school is a demonstration and laboratory experiment, which the student must deal with in the classroom with the teacher's explanations, in laboratory work, in a physical workshop, in a physical circle and at home.

Without experiment there is not and cannot be a rational teaching of physics; one verbal learning physics inevitably leads to formalism and rote learning.

An experiment in a school physics course is a reflection of the scientific method of research inherent in physics.

Setting up experiments and observations is of great importance for familiarizing students with the essence of the experimental method, with its role in scientific research in physics, as well as in the formation of skills to independently acquire and apply knowledge, and the development of creative abilities.

The skills formed during the experiments are important aspect to positively motivate students for research activities. In school practice, the experiment, the experimental method and the experimental activities of students are implemented mainly when staging demonstration and laboratory experiments, in problem-search and research teaching methods.

A separate group of experimental foundations of physics is fundamental scientific experiments. A number of experiments are demonstrated on the equipment available at the school, others - on models, and still others - by watching movies. The study of fundamental experiments makes it possible to intensify the activity of students, contributes to the development of their thinking, arouses interest, encourages independent research.

A large number of observations and demonstrations does not provide students with the ability to independently and holistically conduct observation. This fact can be related to the fact that in most of the experiments offered to students, the composition and sequence of all operations are determined. This problem has been further exacerbated by the introduction of printed lab notebooks. Students, having completed more than thirty laboratory works on such notebooks only for three years of study (from 9th to 11th grades), cannot determine the main operations of the experiment. Although for students with low and satisfactory learning levels, they provide a situation of success and create cognitive interest, positive motivation. This is once again confirmed by studies: more than 30% of schoolchildren love physics lessons for the opportunity to independently perform laboratory and practical work.

In order for students to form all the elements of experimental methods of educational research in lessons and laboratory work: measurements, observations, fixing their results, conducting mathematical processing of the results obtained, and at the same time their implementation was accompanied by a high degree independence and efficiency, before the start of each experiment, students are offered the heuristic prescription “I am learning to experiment”, and before the observation, the heuristic prescription “I am learning to observe”. They tell students what to do (but not how) they outline the direction of movement forward.

Great opportunities for organizing independent experiments of students have a "Notebook for experimental research of students in grades 10" (authors N.I. Zaprudsky, A.L. Karpuk). Depending on the abilities of students, they are offered two options for conducting (independently using general recommendations for planning and conducting an experiment - option A or in accordance with those proposed in option B step by step). The choice of experimental research and experimental tasks additional to the program provides great opportunities for realizing the interests of students.

In general, in the process of independent experimental activity, students acquire the following specific skills:

· observe and study the phenomena and properties of substances and bodies;

· describe the results of observations;

· put forward hypotheses;

· select the instruments necessary for the experiments;

· take measurements;

· calculate errors of direct and indirect measurements;

· present measurement results in the form of tables and graphs;

· interpret the results of experiments;

draw conclusions;

· discuss the results of the experiment, participate in the discussion.

Educational physical experiment is an integral, organic part of the physics course high school. A successful combination of theoretical material and experiment gives, as practice shows, the best pedagogical result.


.2 Analysis of programs and textbooks on the use of experimental tasks in the school physics course


In high school (grades 10-11), five teaching materials are distributed and used mainly.

UMK - "Physics 10-11" ed. Kasyanov V.A.

Class. 1-3 hours per week. Textbook, ed. Kasyanov V.A.

The course is intended for students of general education classes for whom physics is not a core subject and should be studied in accordance with the basic component curriculum. The main goal is to form schoolchildren's ideas about the methodology of scientific knowledge, the role, place and relationship of theory and experiment in the process of cognition, their relationship, the structure of the Universe and the position of man in the world around him. The course is designed to form an opinion among students about the general principles of physics and the main tasks that it solves; implement environmental education schoolchildren, i.e. form their understanding of the scientific aspects of environmental protection; develop a scientific approach to the analysis of newly discovered phenomena. This teaching material in terms of content and methodology of presentation of educational material has been finalized by the author to a greater extent than others, but it requires 3 or more hours per week (10-11 cells) to study. The kit includes:

Methodological guide for the teacher.

Notebook for laboratory work for each of the textbooks.

UMK - "Physics 10-11", ed. Myakishev G.Ya., Bukhovtsev B.B., Sotsky N.N.

Class. 3-4 hours a week. Textbook, ed. Myakishev G.Ya., Bukhovtsev B.B., Sotsky N.N.

Class. 3-4 hours per week. Textbook, ed. Myakishev G.Ya., Bukhovtsev B.B.

Physics grade 10. Designed for 3 or more hours a week, to the team of the first two well-known authors Myakishev G.Ya., Bukhovtsev B.B. Sotsky N.N. was added, who wrote the section of mechanics, the study of which has now become necessary in the senior profile school. Physics grade 11. 3 - 4 hours a week. The team of authors is the same: Myakishev G.Ya., Bukhovtsev B.B. This course has been little revised, compared to the "old Myakishev" it has not changed much. There is a slight transfer of individual parts in graduation class. This set is a revised version of traditional textbooks (almost the entire USSR studied from them) for high school the same authors.

UMK - "Physics 10-11", ed. Antsiferov L. I.

Class. 3 hours per week. Textbook, ed. Antsiferov L.I.

The course program is based on the cyclic principle of constructing educational material, which provides for the study physical theory, its use in solving problems, the application of theory in practice. Two levels of educational content are distinguished: a basic minimum, which is mandatory for everyone, and educational material of increased difficulty, addressed to schoolchildren who are especially interested in physics. This textbook was written by a well-known methodologist from Kursk prof. Antsiferov L.I. Many years of work in a pedagogical university and lecturing students led to the creation of this school course. These textbooks are difficult for the general education level, require revision and additional teaching materials.

UMK - "Physics 10-11", ed. Gromov S.V.

Class. 3 hours per week. Textbook, ed. Gromov S.V.

Class. 2 hours per week. Textbook, ed. Gromov S.V.

The textbooks are intended for senior classes of secondary schools. Include a theoretical presentation of "school physics". At the same time, considerable attention is paid to historical materials and facts. The order of presentation is unusual: mechanics ends with the head of SRT, followed by electrodynamics, MKT, the quantum physics, physics atomic nucleus and elementary particles. Such a structure, according to the author of the course, makes it possible to form in the minds of students a more rigorous idea of ​​the modern physical picture of the world. The practical part is represented by descriptions of the minimum number of standard laboratory work. The passage of material suggests a decision a large number problems, algorithms for solving their main types are given. In all the above textbooks for high school, the so-called general education level should be implemented, but this will largely depend on the pedagogical skill of the teacher. All these textbooks in a modern school may well be used in classes of natural science, technical, and other profiles, with a grid of 4-5 hours a week.

UMK - "Physics 10-11", ed. Mansurov A. N., Mansurov N. A.

Grade 11. 2 hours (1 hour) per week. Textbook, ed. Mansurov A. N., Mansurov N. A.

Single schools work on this set! But it is the first textbook for the supposed liberal arts of physics. The authors have tried to form an idea of ​​the physical picture of the world; the mechanical, electrodynamic and quantum-statistical pictures of the world are considered sequentially. The content of the course includes elements of methods of cognition. The course contains a fragmentary description of laws, theories, processes and phenomena. The mathematical apparatus is almost not used and has been replaced verbal description physical models. Solving problems and conducting laboratory work is not provided. In addition to the textbook published teaching aids and planning.


3 A new approach to conducting experimental tasks in physics using Lego-constructors on the example of the section "Mechanics"

physics school experimental mechanics

Implementation modern requirements to the formation of experimental skills is impossible without the use of new approaches to practical work. It is necessary to use a methodology in which laboratory work does not perform an illustrative function for the material being studied, but is a full part of the content of education and requires the use of research methods in teaching. At the same time, the role of the frontal experiment increases when studying new material using a research approach, and the maximum number of experiments should be transferred from the teacher's demonstration table to the students' desks. When planning the educational process, it is necessary to pay attention not only to the number of laboratory works, but also to the types of activities that they form. It is desirable to transfer part of the work from carrying out indirect measurements to research on checking the dependencies between quantities and plotting graphs of empirical dependencies. At the same time, pay attention to the formation of the following skills: to design an experimental setup based on the formulation of the experimental hypothesis; build graphs and calculate the values ​​of physical quantities on them; analyze the results of experimental studies, expressed in the form of experimental studies, expressed in the form of a table or graph, draw conclusions from the results of the experiment.

The federal component of the state educational standard in physics assumes the priority of an activity approach to the learning process, the development of students' skills to make observations of natural phenomena, describe and generalize the results of observations, use simple measuring instruments to study physical phenomena; present the results of observations using tables, graphs and identify on this basis empirical dependencies; apply the acquired knowledge to explain various natural phenomena and processes, the principles of operation of the most important technical devices, to solve physical problems. Use in educational process Lego technology is of great importance for the realization of these requirements.

The use of Lego-constructors increases the motivation of students to learn, because. it requires knowledge from almost all academic disciplines from art and history to mathematics and natural sciences. Interdisciplinary classes are based on a natural interest in the design and construction of various mechanisms.

Modern organization learning activities requires that students give theoretical generalizations based on the results of their own activities. For the subject "physics" is a learning experiment.

The role, place and functions of an independent experiment in teaching physics have fundamentally changed: students must master not only specific practical skills, but also the basics natural scientific method knowledge, and this can only be realized through a system of independent experimental research. Lego-constructors significantly mobilize such research.

A feature of teaching the subject "Physics" in 2009/2010 academic year is the use of educational Lego - designers, which allow you to fully implement the principle of student-centered learning, conduct demonstration experiments and laboratory work, covering almost all topics of the physics course and performing not so much an illustrative function for the material being studied, but requiring the use of research methods, which contributes to increasing interest in the subject being studied.

1.Entertainment industry. PervoRobot. Includes: 216 LEGO elements including RCX block and IR transmitter, ambient light sensor, 2 touch sensors, 2 9V motors.

2.automated devices. PervoRobot. Includes: 828 Lego bricks including RCX Lego computer, infrared transmitter, 2 light sensors, 2 touch sensors, 2 9V motors.

.FirstRobot NXT. The set includes: a programmable NXT control unit, three interactive servomotors, a set of sensors (distance, touch, sound, light, etc.), a battery, connecting cables, as well as 407 constructive LEGO elements - beams, axles, gears, pins, bricks , plates, etc.

.Energy, work, power. Contents: Four identical, fully stocked mini-kits of 201 parts each, including motors and electrical capacitors.

.Technology and physics. The set contains: 352 parts designed to study the basic laws of mechanics and the theory of magnetism.

.Pneumatics. The kit includes pumps, pipes, cylinders, valves, an air reservoir and a pressure gauge for building pneumatic models.

.Renewable energy sources. In the set: 721 elements, including a micromotor, solar battery, various gears and connecting wires.

PervoRobot kits based on RCX and NXT control units are designed to create programmable robotic devices that allow collecting data from sensors and their primary processing.

Educational Lego-constructors of the "EDUCATIONAL" series (education) can be used in the study of the "Mechanics" section (blocks, levers, types of movement, energy transformation, conservation laws). With sufficient motivation and methodological preparation, with the help of Lego thematic kits, it is possible to cover the main sections of physics, which will make classes interesting and effective, and, therefore, provide high-quality training for students.


.4 Methodology for conducting a pedagogical experiment at the level of ascertaining experiment


There are two options for constructing a pedagogical experiment.

The first - when two groups of children participate in the experiment, one of which is engaged in an experimental program, and the second - in a traditional one. At the third stage of the study, the levels of knowledge and skills of both groups will be compared.

The second is when one group of children participates in the experiment, and at the third stage the level of knowledge before and after the formative experiment is compared.

In accordance with the hypothesis and objectives of the study, a plan of a pedagogical experiment was developed, which included three stages.

The ascertaining stage was carried out in a month, a year. Its purpose was to study the features / knowledge / skills, etc. ... in children ... of age.

At the formative stage (month, year), work was carried out to form ..., using ....

The control stage (month, year) aimed at checking the assimilation of children ... age pilot program knowledge/skills.

The experiment was conducted in .... The number of children participated in it (indicate age).

At the first stage of the ascertaining experiment, the ideas / knowledge / skills of children about ....

A series of tasks was developed to study the knowledge of children....

exercise. Target:

Analysis of the assignment showed: ...

exercise. Target:

Task performance analysis...

exercise. ...

From 3 to 6 tasks.

The results of task analysis should be placed in tables. The tables indicate the number of children or the percentage of their total number. The tables can indicate the levels of development of a given skill in children, or the number of completed tasks, etc. Table example:


Table no....

Number of children No. No. Absolute number% 1 task (for certain knowledge, skills) 2 task 3 task

Or such a table: (in this case, it is necessary to indicate by what criteria children belong to a particular level)

To identify the level of ... in children, we developed the following criteria:

Three levels have been identified....:

Tall: ...

Average: ...

Short: ...

Table No. shows the ratio of the number of children in the control and experimental groups by levels.


Table no....

Level of knowledge/skillsNumber of children №№Absolute number%HighAverageLow

The data obtained indicate that...

The experimental work carried out made it possible to determine the ways and means ... .


1.5 Conclusions on the first chapter


In the first chapter, we considered the role and significance of experimental tasks in the study of physics at school. Definitions are given: experiment in pedagogy, psychology, philosophy, methods of teaching physics, experimental tasks in the same areas.

After analyzing all the definitions, we can draw the following conclusion about the essence of the experimental tasks. Of course, the definition of these tasks as research tasks is somewhat arbitrary, since the possibility of a school physics classroom and the level of preparedness of students even in high school make the task of conducting physical research impossible. Therefore, research, creative tasks should include those tasks in which the student can discover new patterns unknown to him or for the solution of which he must make some inventions. Such an independent discovery of a law known in physics or the invention of a method for measuring a physical quantity is not a simple repetition of the known. This discovery or invention, which has only a subjective novelty, is for the student an objective proof of his ability for independent creativity, allows him to acquire the necessary confidence in his strengths and abilities. And yet it is possible to solve this problem.

After analyzing the programs and textbooks "Physics" Grade 10 on the use of experimental tasks in the "Mechanics" section. It can be said that laboratory work and experiments in this course are not enough to fully perceive all the material in the "Mechanics" section.

A new approach to teaching physics is also considered - the use of Lego - constructors that allow developing the creative thinking of students.


Chapter 2


1 Development of systems of experimental tasks on the topic "Kinematics of a point". Methodological recommendations for use in physics lessons


13 hours are allotted to study the topic of point kinematics.

Movement with constant acceleration.

An experimental task has been developed for this topic:

An Atwood machine is used to do the job.

To perform the work, the Atwood machine must be installed strictly vertically, which is easy to check by the parallelism of the scale and the thread.

Purpose of the experiment: Verification of the law of speeds

measurements

Check the verticality of the Atwood machine. Balancing loads.

The annular shelf P1 is fixed on the scale. Adjust its position.

Impose on the right load overloads in 5-6 g.

Moving uniformly accelerated from the upper position to the annular ledge, the right-hand load travels the path S1 in time t1 and acquires speed v by the end of this movement. On the annular shelf, the load relieves overloads and then moves evenly at the speed that it acquired at the end of acceleration. To determine it, it is necessary to measure the time t2 of the movement of the load on the path S2. Thus, each experiment consists of two measurements: first, the time of uniformly accelerated movement t1 is measured, and then the load is re-launched to measure the time uniform motion t2.

Carry out 5-6 experiments with different values path S1 (in increments of 15-20 cm). Path S2 is chosen arbitrarily. The data obtained is entered into the report table.

Methodological features:

Despite the fact that the basic equations of kinematics rectilinear motion have a simple form and are not in doubt, the experimental verification of these relationships is very difficult. The difficulty arises mainly for two reasons. First, at sufficiently high velocities of motion of bodies, it is necessary to measure the time of their motion with great accuracy. Secondly, friction and resistance forces act in any system of moving bodies, which are difficult to take into account with a sufficient degree of accuracy.

Therefore, it is necessary to carry out such experiments and experiments that remove all difficulties.


2 Development of systems of experimental tasks on the topic "Rigid Body Kinematics". Methodological recommendations for use in physics lessons


The study of the topic Kinematics takes 3 hours, and includes the following sections:

mechanical movement and its relativity. Translational and rotational motion of a rigid body. Material point. Trajectory of movement. Uniform and uniformly accelerated movement. Free fall. The movement of the body in a circle. On this topic, we proposed the following experimental task:

Objective

Experimental verification of the basic equation of dynamics rotary motion rigid body around a fixed axis.

Experiment Idea

The experiment investigates the rotational motion of a system of bodies fixed on an axis, in which the moment of inertia can change (Oberbeck's pendulum). Various moments external forces are created by weights suspended from a thread wound around a pulley.

Experimental setup

The axis of the Oberbeck pendulum is fixed in bearings so that the whole system can rotate around a horizontal axis. By moving the weights along the spokes, you can easily change the moment of inertia of the system. A thread is wound onto the pulley turn to turn, to which the platform is attached known mass. Weights from the set are superimposed on the platform. The height of the fall of the goods is measured using a ruler, parallel to the thread. The Oberbeck pendulum can be equipped with an electromagnetic clutch - a starter and an electronic stopwatch. Before each experiment, the pendulum should be carefully adjusted. Special attention it is necessary to pay attention to the symmetry of the location of the goods on the cross. In this case, the pendulum is in a state of indifferent equilibrium.

Conducting an experiment

Task 1. Estimation of the torque of the friction force acting in the system

measurements

Install the weights m1 on the cross in the middle position, placing them at an equal distance from the axis so that the pendulum is in a position of indifferent equilibrium.

By imposing small loads on the platform, one determines approximately the minimum mass m0 at which the pendulum starts to rotate. Estimate the moment of friction force from the ratio

where R is the radius of the pulley on which the thread is wound.

It is desirable to carry out further measurements with weights m 10m0.

Task 2. Verification of the basic equation of the dynamics of rotational motion

measurements

Strengthen the loads m1 at a minimum distance from the axis of rotation. Balance the pendulum. Measure the distance r from the axis of the pendulum to the centers of the weights.

Wind the thread around one of the pulleys. On the scale bar choose the initial position of the platform, making a count, for example, along its bottom edge. Then the final position of the load will be at the level of the raised receiving platform. The drop height h is equal to the difference between these readings and can be left the same in all experiments.

Place the first load on the platform. Having placed the load at the level of the upper reference, this position is fixed by clamping the thread with an electromagnetic clutch. Prepare an electronic stopwatch for measurement.

The thread is released, allowing the load to fall. This is achieved by disengaging the clutch. This automatically starts the stopwatch. Hitting the receiving platform stops the fall of the load and stops the stopwatch.

The fall time measurement with the same load is performed at least three times.

Carry out measurements of the time of the fall of the load m at other values ​​of the moment Mn. To do this, either additional overloads are added to the platform, or the thread is transferred to another pulley. With the same value of the moment of inertia of the pendulum, it is necessary to carry out measurements with at least five values ​​of the moment Mn.

Increase the moment of inertia of the pendulum. To do this, it is sufficient to symmetrically move the loads m1 by several centimeters. The step of such a movement should be chosen in such a way as to obtain 5-6 values ​​of the moment of inertia of the pendulum. Carry out measurements of the time of the fall of the load m (p. 2-p. 7). All data is entered in the report table.


3 Development of systems of experimental tasks on the topic "Dynamics". Methodological recommendations for use in physics lessons


18 hours are allotted for studying the topic Dynamics.

Resistance forces during the motion of solid bodies in liquids and gases.

Purpose of the experiment: To show how air speed affects the flight of an aircraft.

Materials: small funnel, table tennis ball.

Turn the funnel upside down.

Insert the ball into the funnel and support it with your finger.

Blow into the narrow end of the funnel.

Stop supporting the ball with your finger, but keep blowing.

Results: The ball remains in the funnel.

Why? The faster air passes by the ball, the less pressure it exerts on the ball. The air pressure above the ball is much less than below it, so the ball is supported by the air below it. Due to the pressure of the moving air, the wings of the aircraft are pushed up, as it were. Due to the shape of the wing, air moves faster above its upper surface than under its lower surface. Therefore, there is a force that pushes the plane up - lift. .


4 Development of systems of experimental tasks on the topic "Conservation laws in mechanics". Methodological recommendations for use in physics lessons


On the topic of conservation laws in mechanics, 16 hours are allotted.

Law of conservation of momentum. (5 o'clock)

For this topic, we proposed the following experimental task:

Purpose: study of the law of conservation of momentum.

Each of you probably faced such a situation: you run at a certain speed along the corridor and collide with a standing person. What is happening to this person? Indeed, he begins to move, i.e. gains speed.

Let's do an experiment on the interaction of two balls. Two identical balls hang on thin threads. Let's move the left ball aside and let it go. After the collision of the balls, the left one will stop, and the right one will start moving. The height to which the right ball will rise will coincide with that to which the left ball was deflected before. That is, the left ball transfers all its momentum to the right one. By how much the momentum of the first ball decreases, the momentum of the second ball will increase by the same amount. If we talk about a system of 2 balls, then the momentum of the system remains unchanged, that is, it is preserved.

Such a collision is called elastic (slides No. 7-9).

Signs of elastic impact:

-There is no permanent deformation and therefore both conservation laws in mechanics are satisfied.

-Bodies after interaction move together.

-Examples of this type of interaction: playing tennis, hockey, etc.

-If the mass of the moving body is greater than the mass of the stationary one (m1 > m2), then it reduces the speed without changing direction.

-If vice versa, then the first body is reflected from it and moves in the opposite direction.

There is also an inelastic collision

Let's observe: take one big ball, one small one. The small ball is at rest, and the large one is set in motion towards the small one.

After the collision, the balls move together at the same speed.

Signs of elastic impact:

-As a result of the interaction, the bodies move together.

-The bodies have residual deformation, therefore, mechanical energy is converted into internal energy.

-Only the law of conservation of momentum is satisfied.

-Examples from life experience: a meteorite colliding with the Earth, hitting an anvil with a hammer, etc.

-With equal masses (one of the bodies is motionless), half of the mechanical energy is lost,

-If m1 is much less than m2, then it is lost most of(bullet and wall)

-If, on the contrary, an insignificant part of the energy is transferred (an icebreaker and a small ice floe).

That is, there are two types of collisions: elastic and inelastic. .


5 Development of systems of experimental tasks on the topic "Statics". Methodological recommendations for use in physics lessons


On the study of the topic “Static. Equilibrium of absolutely solid bodies” is given 3 hours.

For this topic, we proposed the following experimental task:

The purpose of the experiment: Find the position of the center of gravity.

Materials: plasticine, two metal forks, a toothpick, a tall glass or a jar with a wide mouth.

Roll the plasticine into a ball with a diameter of about 4 cm.

Insert a fork into the ball.

Insert the second fork into the ball at an angle of 45 degrees with respect to the first fork.

Insert a toothpick into the ball between the forks.

Place the toothpick with the end on the edge of the glass and move towards the center of the glass until balance is reached.

Results: At a certain position of the toothpick, the forks are balanced.

Why? Since the forks are located at an angle to each other, their weight is, as it were, concentrated at a certain point of the stick located between them. This point is called the center of gravity.


.6 Conclusions on the second chapter


In the second chapter, we presented experimental tasks on the topic "Mechanics".

It was found that each experiment, the development of concepts that allow qualitative characteristics in the form of a number. In order to draw general conclusions from observations, to find out the causes of phenomena, it is necessary to establish quantitative relationships between quantities. If such a dependence is obtained, then a physical law is found. If a physical law is found, then there is no need to put in each separate case experience, it is enough to perform the corresponding calculations.

Having studied experimentally the quantitative relationships between the quantities, it is possible to identify patterns. Based on these regularities, a general theory of phenomena is developed.


Conclusion


Already in the definition of physics as a science, there is a combination of both theoretical and practical parts in it. It is considered important that in the process of teaching students physics, the teacher should be able to demonstrate to his students the relationship of these parts as fully as possible. After all, when students feel this relationship, they will be able to give a correct theoretical explanation to many of the processes taking place around them in everyday life, in nature. This may be an indicator of a fairly complete mastery of the material.

What forms of practical training can be offered in addition to the teacher's story? First of all, of course, this is the observation by students of the demonstration of experiments conducted by the teacher in the classroom when explaining new material or when repeating what has been passed, it is also possible to offer experiments conducted by the students themselves in the classroom during lessons in the process of frontal laboratory work under the direct supervision of the teacher. You can also suggest: 1) experiments conducted by the students themselves in the classroom during a physical workshop; 2) experiments-demonstrations conducted by students when answering; 3) experiments conducted by students outside the school on the teacher's homework; 4) observations of short-term and long-term phenomena of nature, technology and everyday life, carried out by students at home on special assignments from the teacher.

Experience not only teaches, it captivates the student and makes him better understand the phenomenon that he demonstrates. After all, it is known that a person interested in the final result achieves success. So in this case, having interested the student, we will awaken the craving for knowledge.


Bibliography


1.Bludov M.I. Conversations on physics. - M.: Enlightenment, 2007. -112 p.

2.Burov V.A. et al. Frontal experimental tasks in physics in high school. - M.: Academy, 2005. - 208 p.

.Gallinger I.V. Experimental assignments in physics lessons // Physics at school. - 2008. - No. 2. - S. 26 - 31.

.Znamensky A.P. Fundamentals of physics. - M.: Enlightenment, 2007. - 212 p.

5.Ivanov A.I. and others. Frontal experimental tasks in physics: for the 10th grade. - M.: Vuzovsky textbook, 2009. - 313 p.

6.Ivanova L.A. Activation of cognitive activity of students in physics lessons when studying new material. - M.: Enlightenment, 2006. - 492 p.

7.Research in psychology: methods and planning / J. Goodwin. St. Petersburg: Piter, 2008. - 172 p.

.Kabardin O.F. Pedagogical experiment// Physics at school. - 2009. - No. 6. - S. 24-31.

9.Myakishev G.Ya., Bukhovtsev B.B., Sotsky N.N. Physics. Grade 10. Textbook: Textbook. - M.: Gardarika, 2008. - 138 p.

10.Programs for educational institutions. Physics. Compiled by Yu.I. Dick, V.A. Korovin. - M.: Enlightenment, 2007. -112 p.

11.Rubinshtein S.L. Fundamentals of psychology. - M.: Enlightenment, 2007. - 226 p.

.Slastenin V. Pedagogy. - M.: Gardariki, 2009. - 190 p.

.Sokolov V.V. Philosophy. - M.: Higher school, 2008. - 117 p.

14.Theory and methods of teaching physics at school. General issues. Under the editorship of S.E. Kamenetsky, N.S. Purysheva. - M.: GEOTAR Media, 2007. - 640 p.

15.Kharlamov I.F. Pedagogy. Ed. 2nd revision and additional - M.: Higher School, 2009 - 576s.

16.Shilov V.F. Home experimental tasks in physics. 9 - 11 classes. - M.: Knowledge, 2008. - 96 p.

Answer to the question


The relationship between the real and the possible, the relationship between there is and may be - this is the intellectual innovation that, according to the classical studies of J. Piaget and his school, becomes available to children after 11-12 years. Numerous critics of Piaget tried to show that the age of 11-12 years is very conditional and can be shifted in any direction, that the transition to a new intellectual level is not a jerk, but goes through a number of intermediate stages. But no one disputed the very fact that a new quality appears in the intellectual life of a person on the border of primary school and adolescence. The adolescent begins his analysis of the problem with an attempt to find out the possible relations that apply to the data at his disposal, and then tries, by a combination of experiment and logical analysis, to establish which of the possible relations really exist here.

A fundamental reorientation of thinking from knowing how reality works to finding potential that lie behind the immediate given is called the transition to hypothetical-deductive thinking.

New hypothetical-deductive means of comprehending the world sharply expand the boundaries of the adolescent's inner life: his world is filled with ideal constructions, hypotheses about himself, those around him, and humanity as a whole. These hypotheses go far beyond the boundaries of existing relationships and directly observable properties of people (including oneself) and become the basis for experimental testing of one's own potentialities.

Hypothetical-deductive thinking is based on the development of combinatorics and propositional operations. The first step of cognitive restructuring is characterized by the fact that thinking becomes less objective and visual. If, at the stage of concrete operations, the child sorts objects only on the basis of identity or similarity, it now becomes possible to classify heterogeneous objects in accordance with arbitrarily chosen criteria of a higher order. New combinations of objects or categories are analyzed, abstract statements or ideas are compared with each other in a variety of ways. Thinking goes beyond observable and limited reality and operates with an arbitrary number of any combinations. By combining objects, it is now possible to systematically cognize the world, to detect possible changes in it, although adolescents are not yet able to express the mathematical laws behind this with formulas. However, the very principle of such a description has already been found and realized.

Propositional operations - mental actions, carried out, unlike concrete operations, not with subject representations, but with abstract concepts. They cover propositions that are combined in terms of their conformity or inconsistency with the proposed situation (true or false). It is not simple new way to link facts, but a logical system, which is much richer and more variable than specific operations. It becomes possible to analyze any situation regardless of the actual circumstances; adolescents for the first time acquire the ability to systematically build and test hypotheses. At the same time, further development of specific mental operations. abstract concepts(such as volume, weight, strength, etc.) are now processed in the mind regardless of the specific circumstances. It becomes possible to reflect on one's own thoughts. It is based on conclusions that no longer need to be verified in practice, since they comply with the formal laws of logic. Thinking begins to obey formal logic.

Thus, between the 11th and 15th years of life, significant structural changes occur in the cognitive area, expressed in the transition to the abstract and formal thinking. They complete the line of development, which began in infancy with the formation of sensorimotor structures and continues in childhood until the prepubertal period, with the formation of specific mental operations.

Laboratory work "Electromagnetic induction"

In this work, the phenomenon of electromagnetic induction is studied.

Work goals

Measure the voltage generated by the movement of the magnet in the coil.

Investigate the effects of changing the poles of a magnet when moving in a coil, changing the speed of moving a magnet, using different magnets on the resulting voltage.

Find change magnetic flux when the magnet is lowered into the coil.

Work order

Place the tube on the coil.

Attach the tube to the tripod.

Connect the voltage sensor to output 1 of the Panel. When working with the CoachLab II/II+ Panel, wires with 4 mm plugs are used instead of a voltage sensor.

Connect the wires to the yellow and black sockets of output 3 (this circuit is shown in the figure and described in the Coach Labs section).

Open Labs Coach 6 Explore Physics > Electromagnetic Induction.

Start measurements by pressing the Start button. When the job is done, automatic recording is used. Thanks to this, despite the fact that the experiment lasts about half a second, it is possible to measure the resulting induction emf. When the amplitude of the measured voltage reaches certain value(by default, when the voltage increases and reaches the value of 0.3 V), the computer will start recording the measured signal.

Start sliding the magnet into the plastic tube.

Measurements will start when the voltage reaches 0.3 V, which corresponds to the beginning of the magnet lowering.

If the minimum value for triggering is very close to zero, then recording may start due to signal interference. Therefore, the minimum value to start should not be close to zero.

If the trigger value is higher than the maximum (lower than the minimum) voltage value, recording will never start automatically. In this case, you need to change the launch conditions.

Data analysis

It may turn out that the obtained dependence of the voltage on time is not symmetrical with respect to the zero value of the voltage. This means there is interference. This will not affect the qualitative analysis, but corrections must be made in the calculations to take into account these interferences.

Explain the waveform (minimums and maxima) of the recorded voltage.

Explain why the highs (lows) are not symmetrical.

Determine when the magnetic flux changes the most.

Determine the total change in magnetic flux during the first half of the moving stage when the magnet was pushed into the coil?

To find this value, use either the Process/Analyze > Area or Process/Analyze > Integral options.

Determine the total change in magnetic flux during the second half of the moving stage when the magnet was pulled out of the coil?


Tags: Development of a system of experimental tasks in physics on the example of the section "Mechanics" Diploma in Pedagogy

The paper presents recommendations, in the form of algorithms, for organizing experiments conducted by the students themselves in the classroom with answers, outside the school on the teacher's homework; on the organization of short-term and long-term observations of natural phenomena, tasks of an inventive nature for the creation of equipment for experiments, operating models of machines and mechanisms carried out by students at home on special tasks of the teacher, the types of physical experiments are also systematized in the work, examples of experimental tasks for different topics and sections of physics grades 7-9.

Download:


Preview:

municipal competition

socially important pedagogical innovations in the field

general, preschool and additional education

municipality of the resort city of Gelendzhik

organization of experimental work

in physics lessons and outside of school hours.

physics and mathematics teacher

MAOU secondary school №12

resort city of Gelendzhik

Krasnodar Territory

Gelendzhik - 2015

Introduction …………………………………………………………………….....3

1.1 Types of physical experiments.……….. …………………………..5

2.1 Algorithm for creating experimental tasks…….……………..8

2.2 Results of testing experimental tasks in grades 7-9 .............................................................. ................................................. ...................ten

Conclusion …………………………………………………………………...12

Literature …………………………………………………………………....13

Appendix………………………………………………………………….14

4. Lesson in the 8th grade on the topic "Serial and parallel

Connection of conductors.

"The joy of seeing and understanding is the most beautiful gift of nature."

Albert Einstein

Introduction

In accordance with the new requirements of the state educational standard, the methodological basis of education is a system-activity approach that allows students to form universal learning activities among which an important place is occupied by the acquisition of experience in the application of scientific methods of cognition, the formation of skills in experimental work.

One of the ways to connect theory with practice is to set up experimental tasks, the solution of which shows students the laws in action, reveals the objectivity of the laws of nature, their obligatory implementation, shows the use by people of knowledge of the laws of nature to predict phenomena and control them, the importance of studying them to achieve specific, practical purposes. Especially valuable should be recognized such experimental problems, the data for the solution of which are taken from the experience taking place before the eyes of the students, and the correctness of the solution is checked by experience or a control device. In this case, the theoretical principles studied in the course of physics acquire special significance in the eyes of students. It is one thing to come to certain conclusions and their mathematical formulation through reasoning and experiment, i.e. to a formula that will have to be learned by heart and be able to deduce, and be limited to this, another thing is to be able to manage them on the basis of these conclusions and formulas.

Relevance innovation is due to the fact that the organization academic work should be set in such a way that it affects the personal sphere of children, and the teacher would create new forms of work. The creative direction of work brings the teacher and the student together, activates the cognitive activity of the participants in the educational process.

The paper presents recommendations in the form of algorithms for organizing experiments conducted by the students themselves in the classroom when answering, outside the school on the teacher's homework; on the organization of observations of short-term and long-term natural phenomena, tasks of an inventive nature for the creation of equipment for experiments, operating models of machines and mechanisms carried out by students at home on special tasks of the teacher, the types of physical experiments are also systematized in the work, examples of experimental tasks on various topics and sections are given physics grades 7-9. The following materials were used in this work. physical experiments used in work on projects, during educational activities and after school hours:

Burov V.

Mansvetova G.P., Gudkova V.F.Physical experiment at school. From work experience. A guide for teachers. Issue 6 / - M .: Education, 1981. - 192s., Ill., as well as materials from the Internethttp://kopilkaurokov.ru/ , http://www.metod-kopilka.ru/ ,

When analyzing similar products existing in Russia have been revealed: in physics, and in the education system as a whole, there have been big changes. The emergence of a new product on this topic will replenish methodical piggy bank physics teachers and intensifies work on the implementation of the Federal State Educational Standard in teaching physics.

All the experiments presented in the work were carried out at physics lessons in grades 7-9 of the Moscow Autonomous Educational Institution Secondary School No. 12, in the process of preparing for the Unified State Exam in physics in grade 11, during the Physics Week, some of them were demonstrated by me at the GMO meeting physics teachers, published on the social networking site of the education workers website.

Chapter I. Place of experiment in the study of physics

  1. Types of physical experiments

The explanatory note to the programs in physics refers to the need to familiarize students with the methods of science.

Methods of physical science are divided into theoretical and experimental. In this paper, the "experiment" is considered as one of the fundamental methods in the study of physics.

The word "experiment" (from the Latin experimentum) means "test", "experience". The experimental method arose in the natural sciences of modern times (G. Galileo, W. Hilbert). His philosophical understanding was first given in the works of F. Bacon.A learning experiment is a means of learning in the form of experiments specially organized and conducted by a teacher and a student.

Objectives of the educational experiment:

  • Solving the main educational tasks;
  • Formation and development of cognitive and mental activity;
  • Polytechnic training;
  • Formation of the scientific outlook of students.

Educational physical experiments can be combined into the following groups:

Demo Experiment, being a means of visualization, contributes to the organization of students' perception of educational material, its understanding and memorization; allows for polytechnic education of students; promotes an increase in interest in the study of physics and the creation of motivation for learning. When demonstrating an experiment, it is important that the students themselves can explain the phenomenon they have seen and come to a common conclusion by brainstorming. I often use this method when explaining new material. I also use video fragments with experiments without sound accompaniment on the topic under study and ask them to explain the observed phenomenon. Then I propose to listen to the soundtrack and find an error in my reasoning.
While doing
laboratory workstudents gain experience of independent experimental activity, they havesuch important personal qualities as accuracy in the work of instruments are developed; observance of cleanliness and order in the workplace, in the records that are made during the experiment, organization, perseverance in obtaining results. They form a certain culture of mental and physical labor.

Home experimental tasks and laboratory workare performed by students at home without direct control from the teacher over the progress of work.
Experimental works of this type form in students:
- the ability to observe physical phenomena in nature and in everyday life;
- the ability to perform measurements using measuring instruments used in everyday life;
- interest in experiment and in the study of physics;
- independence and activity.
In order for the student to be able to spend at home laboratory work the teacher must conduct a detailed briefing and give a clear algorithm of actions to the student.

Experimental tasksare tasks in which students receive data from experimental conditions. According to a special algorithm, students assemble an experimental setup, perform measurements, and use the measurement results to solve the problem.
Creation of operating models of devices, machines and mechanisms. Every year at school, as part of the week of physics, I hold an inventor competition, to which students submit all their inventive ideas. Before the lesson, they demonstrate their invention and explain what physical phenomena and laws underlie this invention. Students very often involve their parents in working on their inventions, and this becomes a kind of family project. This type of work has a great educational effect.

2.1 Algorithm for creating experimental tasks

The main purpose of experimental tasks is to promote the formation of basic concepts, laws, theories in students, the development of thinking, independence, practical skills, including the ability to observe physical phenomena, to perform simple experiments, measurements, handle instruments and materials, analyze the results of the experiment, make generalizations and conclusions.

Students are offered the following algorithm for conducting the experiment:

  1. Formulation and justification of the hypothesis that can be used as the basis for the experiment.
  2. Determining the purpose of the experiment.
  3. Finding out the conditions necessary to achieve the goal of the experiment.
  4. Experiment planning.
  5. Selection of necessary equipment and materials.
  6. Installation collection.
  7. Conducting an experiment, accompanied by observations, measurements and recording their results.
  8. Mathematical processing of measurement results.
  9. Analysis of the results of the experiment, formulation of conclusions.

The general structure of a physical experiment can be represented as:


When conducting any experiment, it is necessary to remember the requirements for the experiment.

Experiment Requirements:

  • visibility;
  • short duration;
  • Persuasiveness, accessibility, reliability;
  • Security.

2.2 Results of testing experimental problems

in grades 7-9

Experimental tasks are tasks that are small in volume, directly related to the material being studied, aimed at mastering practical skills that are included in different stages of the lesson (knowledge testing, learning new educational material, consolidated knowledge, independent work in class). After completing the experimental task, it is very important to analyze the results obtained and draw conclusions.

Consider various forms creative tasks that I used in my work at each individual stage of teaching physics in high school:

In 7th grade acquaintance with physical terms, with physical quantities and methods of studying physical phenomena begins. One of the visual methods for studying physics is experiments that can be done both in the classroom and at home. Here, experimental tasks and creative tasks can be effective, where you need to figure out how to measure physical quantity or how to demonstrate a physical phenomenon. I always appreciate this kind of work.

In 8th grade I use the following forms of experimental tasks:

1) research tasks - as elements of the lesson;

2) experimental homework;

3) make a small report - research on some topics.

In 9th grade the level of complexity of experimental tasks should be higher. Here I am applying:

1) creative tasks for setting up an experiment at the beginning of the lesson - as an element of a problem task; 2) experimental tasks - as a consolidation of the material covered, or as an element of predicting the result; 3) research tasks - as a short-term laboratory work (10-15 minutes).

The use of experimental tasks in the classroom and outside of school hours as homework led to an increase in the cognitive activity of students, increased interest in the study of physics.

I conducted a survey in the 8th grade, in which physics is studied in the second year, and received the following results:

Questions

Answer options

8A class

8B class

  1. rate your attitude to the subject.

a) don't like the subject

b) I'm interested

c) I love the subject, I want to learn more.

2. How often do you study the subject?

a) regularly

b) sometimes

c) very rarely

3. Do you read additional literature by subject?

a) constantly

b) sometimes

c) little, I don’t read at all

4. Do you want to know, understand, get to the bottom of the matter?

a) almost always

b) sometimes

c) very rarely

5. Would you like to do experiments outside of school hours?

a) yes, very

b) sometimes

c) enough lesson

Of the two 8th grades, there were 24 students who wanted to study physics more deeply and engage in experimental work.

Monitoring the quality of student learning

(teacher Petrosyan O.R.)

Participation in Physics Olympiads and competitions for 4 years

Conclusion

“The childhood of a child is not a period of preparation for a future life, but a full life. Therefore, education should be based not on the knowledge that will be useful to him someday in the future, but on what the child urgently needs today, on the problems of his real life.(John Dewey).

Each modern school Russia has the necessary minimum equipment for conducting physical experiments presented in the paper. In addition, home experiments are carried out exclusively from improvised means. The creation of the simplest models and mechanisms does not require large expenses, and students take up the work with great interest, involving their parents. This product is intended for use by secondary school physics teachers.

Experimental tasks provide students with the opportunity to independently identify the root cause of a physical phenomenon through experience in the process of its direct consideration. Using the simplest equipment, even household items, when conducting an experiment, physics in the minds of students from an abstract system of knowledge turns into a science that studies "the world around us." This emphasizes the practical significance of physical knowledge in everyday life. In the lessons with the experiment, there is no flow of information coming only from the teacher, there are no bored, indifferent views of students. Systematic and purposeful work on the formation of the skills and abilities of experimental work makes it possible, already at the initial stage of studying physics, to involve students in scientific research, teach them to express their thoughts, conduct a public discussion, and defend their own conclusions. This means making learning more effective and meeting modern requirements.

Literature

  1. Bimanova G.M. "The use of innovative technologies in teaching physics in high school." Teacher of secondary school No. 173, Kyzylorda-2013 http://kopilkaurokov.ru/
  2. Braverman E.M. Independent conduct of experiments by students // Physics at school, 2000, No. 3 - from 43 - 46.
  3. Burov V. A. et al. Frontal experimental tasks in physics in grades 6-7 of secondary school: A guide for teachers / V.A. Burov, S.F. Kabanov, V.I. Sviridov. - M.: Enlightenment, 1981. - 112 p., ill.
  4. Gorovaya S.V. "Organization of observations and setting up an experiment in a physics lesson is one of the ways to form key competencies." Physics teacher MOU secondary school No. 27, Komsomolsk-on-Amur-2015

Appendix

Methodological development of physics lessons in grades 7-9 with experimental tasks.

1. Lesson in the 7th grade on the topic "Pressure of solids, liquids and gases."

2. Lesson in the 7th grade on the topic "Solving problems to determine the efficiency of the mechanism."

3. Lesson in the 8th grade on the topic “Thermal phenomena. Melting and solidification".

4. Lesson in the 8th grade on the topic "Electrical Phenomena".

5. Lesson in the 9th grade on the topic "Newton's Laws".

A learning experiment is a means of learning in the form of experiments specially organized and conducted by a teacher and a student. Objectives of the educational experiment: Solving the main educational tasks; Formation and development of cognitive and mental activity; Polytechnic training; Formation of the scientific outlook of students. "The joy of seeing and understanding is the most beautiful gift of nature." Albert Einstein

Experimental tasks Creation of operating models, devices, machines and mechanisms Home experimental tasks Laboratory work Demonstration experiment Physical experiment Educational physical experiments can be grouped into the following groups:

The demonstration experiment, being a means of visualization, contributes to the organization of students' perception of educational material, its understanding and memorization; allows for polytechnic education of students; promotes an increase in interest in the study of physics and the creation of motivation for learning. When demonstrating an experiment, it is important that the students themselves can explain the phenomenon they have seen and come to a common conclusion by brainstorming. I often use this method when explaining new material. I also use video fragments with experiments without sound accompaniment on the topic under study and ask them to explain the observed phenomenon. Then I propose to listen to the soundtrack and find an error in my reasoning.

When performing laboratory work, students gain experience in independent experimental activities, they develop such important personal qualities as accuracy in working with devices; observance of cleanliness and order in the workplace, in the records that are made during the experiment, organization, perseverance in obtaining results. They form a certain culture of mental and physical labor.

Home experimental tasks and laboratory work are carried out by students at home without direct control from the teacher over the progress of work. Experimental works of this type form in students: - the ability to observe physical phenomena in nature and in everyday life; - the ability to perform measurements using measuring instruments used in everyday life; - interest in experiment and in the study of physics; - independence and activity. In order for the student to conduct laboratory work at home, the teacher must conduct a detailed briefing and give a clear algorithm of actions to the student.

Experimental tasks are tasks in which students obtain data from experimental conditions. According to a special algorithm, students assemble an experimental setup, perform measurements, and use the measurement results to solve the problem.

Creation of operating models of devices, machines and mechanisms. Every year at school, as part of the week of physics, I hold an inventor competition, to which students submit all their inventive ideas. Before the lesson, they demonstrate their work and explain what physical phenomena and laws underlie this invention. Students very often involve their parents in the work, and this becomes a kind of family project. This type of work has a great educational effect.

Observation Measurement and recording of results Theoretical analysis and mathematical processing of measurement results Conclusions Structure of a physical experiment

When conducting any experiment, it is necessary to remember the requirements for the experiment. Requirements for the experiment: Visualization; short duration; Persuasiveness, accessibility, reliability; Security.

The use of experimental tasks in the classroom and outside of school hours as homework led to an increase in the cognitive activity of students, increased interest in the study of physics. Questions Answer options Grade 8A Grade 8B Assess your attitude to the subject. a) I don't like the subject, 5% 4% b) I'm interested, 85% 68% c) I like the subject, I want to know more. 10% 28% 2. How often do you study the subject? a) regularly 5% 24% b) sometimes 90% 76% c) very rarely 5% 0% 3. Do you read additional literature on the subject? a) constantly 10% 8% b) sometimes 60% 63% c) little, I don't read at all 30% 29% 4. Do you want to know, understand, get to the bottom of the matter? a) almost always 40% 48% b) sometimes 55% 33% c) very rarely 5% 19% 5. Would you like to do experiments outside of school hours? a) yes, very much 60% 57% b) sometimes 20% 29% c) enough lesson 20% 14%

Monitoring the quality of student learning (teacher Petrosyan O.R.)

Participation in Olympiads and competitions in physics for 4 years

“The childhood of a child is not a period of preparation for a future life, but a full life. Consequently, education should be based not on the knowledge that will be useful to him someday in the future, but on what the child urgently needs today, on the problems of his real life ”(John Dewey). Systematic and purposeful work on the formation of the skills and abilities of experimental work makes it possible, already at the initial stage of studying physics, to involve students in scientific research, teach them to express their thoughts, conduct a public discussion, and defend their own conclusions. This means making learning more effective and meeting modern requirements.

"Be pioneers yourself, explorers! If you don't have a spark, you'll never light it in others!" Sukhomlinsky V.A. Thank you for your attention!



Vibrations and waves.
Optics.

Tasks for independent work.
Task 1. Hydrostatic weighing.
Equipment: wooden ruler length 40 cm, plasticine, a piece of chalk, a measuring cup with water, threads, a razor blade, a tripod with a holder.
Exercise.
Measure

  • plasticine density;
  • chalk density;
  • mass of wooden ruler.

Notes:

  1. It is advisable not to wet a piece of chalk - it can fall apart.
  2. The density of water is considered equal to 1000 kg / m 3

Problem 2. Specific heat of dissolution of hyposulfite.
When dissolving hyposulfite in water, the temperature of the solution decreases greatly.
Measure the specific heat of solution of the given substance.
The specific heat of dissolution is understood as the amount of heat required to dissolve a unit mass of a substance.
The specific heat capacity of water is 4200 J/(kg × K), the density of water is 1000 kg/m 3 .
Equipment: calorimeter; beaker or measuring cup; scales with weights; thermometer; crystalline hyposulfite; warm water.

Problem 3. Mathematical pendulum and free fall acceleration.

Equipment: a tripod with a foot, a stopwatch, a piece of plasticine, a ruler, a thread.
Exercise: Measure free fall acceleration with a mathematical pendulum.

Problem 4. The refractive index of the lens material.
Exercise: Measure the refractive index of the glass that the lens is made from.

Equipment: a biconvex lens on a stand, a light source (a light bulb on a stand with a current source and connecting wires), a screen on a stand, a caliper, a ruler.

Problem 5. "Vibrations of the rod"

Equipment: a tripod with a foot, a stopwatch, a knitting needle, an eraser, a needle, a ruler, a plastic cork from a plastic bottle.

  • Explore the dependence of the oscillation period of the resulting physical pendulum on the length of the upper part of the spoke. Plot the resulting dependency. Check the feasibility of formula (1) in your case.
  • Determine with the maximum possible accuracy the minimum period of oscillation of the resulting pendulum.
  • Determine the value of the free fall acceleration.

Task 6. Determine with the greatest possible accuracy the resistance of the resistor.
Equipment: a current source, a resistor with a known resistance, a resistor with an unknown resistance, a cup (glass, 100 ml), a thermometer, a watch (you can use your wrist), graph paper, a piece of foam.

Task 7. Determine the friction coefficient of the bar on the table.
Equipment: bar, ruler, tripod, threads, weight of known mass.

Task 8. Determine the weight of a flat figure.
Equipment: flat figure, ruler, weight.

Problem 9. Investigate the dependence of the speed of the jet flowing out of the vessel on the height of the water level in this vessel .
Equipment: tripod with clutch and foot, glass burette with scale and rubber tube; spring clip; screw clamp; stopwatch; funnel; cuvette; a glass of water; sheet of graph paper.

Task 10. Determine the temperature of water at which its density is maximum.
Equipment: a glass of water, at a temperature t = 0 °С; metal stand; thermometer; spoon; clock; small glass.

Task 11. Determine the strength of the gap T threads, mg< T .
Equipment: bar whose length 50 cm; thread or thin wire; ruler; cargo of a known mass; tripod.

Task 12. Determine the coefficient of friction of a metal cylinder, the mass of which is known, on the surface of the table.
Equipment: two metal cylinders of approximately the same mass (the mass of one of them is known ( m = 0.4 - 0.6 kg)); length ruler 40 - 50 cm; Bakushinsky dynamometer.

Problem 13. Explore the contents of the mechanical "black box". Determine the characteristics of the rigid body enclosed in the "box".
Equipment: dynamometer, ruler, graph paper, "black box" - a closed jar, partially filled with water, in which there is a solid body with a rigid wire attached to it. The wire exits the can through a small hole in the lid.

Problem 14. Determine the density and specific heat of an unknown metal.
Equipment: calorimeter, plastic cup, bath for developing photographs, measuring cylinder (beaker), thermometer, thread, 2 cylinders of unknown metal, a vessel with hot ( t g \u003d 60 ° -70 °) and cold ( t x \u003d 10 ° - 15 °) with water. Specific heat capacity of water c in \u003d 4200 J / (kg × K).

Problem 15. Determine Young's modulus of steel wire.
Equipment: tripod with two legs for attaching equipment; two steel rods; steel wire (diameter 0.26mm); ruler; dynamometer; plasticine; pin.
Note. The stiffness coefficient of the wire depends on Young's modulus and geometric dimensions wire as follows k = ES/l, where l is the length of the wire, a S is the area of ​​its cross section.

Task 16. Determine the concentration table salt in the aqueous solution given to you.
Equipment: glass jar volume 0.5 l; vessel with aqueous solution table salt of unknown concentration; alternating current source with adjustable voltage; ammeter; voltmeter; two electrodes; connecting wires; key; a set of 8 weights of table salt; graph paper; fresh water container.

Task 17. Determine the resistance of a millivoltmeter and a milliammeter for two measurement ranges.
Equipment: millivoltmeter ( 50/250 mV), milliammeter ( 5/50 mA), two connecting wires, copper and zinc plates, pickles.

Problem 18. Determine the density of the body.
Equipment: an irregular body, a metal rod, a ruler, a tripod, a vessel with water, a thread.

Task 19. Determine the resistances of the resistors R 1, ..., R 7, ammeter and voltmeter.
Equipment: battery, voltmeter, ammeter, connecting wires, switch, resistors: R 1 - R 7.

Problem 20. Determine the coefficient of spring stiffness.
Equipment: spring, ruler, sheet of graph paper, bar, weight 100 g.
Attention! Do not hang a load on the spring, as this will exceed the elastic limit of the spring.

Task 21. Determine the sliding friction coefficient of the match head on the rough surface of the matchbox.
Equipment: box of matches, dynamometer, weight, sheet of paper, ruler, thread.

Problem 22. The part of the fiber optic connector is a glass cylinder (refractive index n= 1.51), which has two round cylindrical channels. The ends of the part are sealed. Determine channel spacing.
Equipment: connector detail, graph paper, magnifier.

Problem 23. "Black vessel". A body is lowered into a "black vessel" with water on a thread. Find the density of the body ρ m , its height l the water level in the vessel with the submerged body ( h) and when the body is outside the liquid ( h o).
Equipment. "Black Vessel", dynamometer, graph paper, ruler.
Density of water 1000 kg/m3. Vessel depth H = 32 cm.

Problem 24. Friction. Determine the sliding friction coefficients of wooden and plastic rulers on the surface of the table.
Equipment. Tripod with foot, plumb line, wooden ruler, plastic ruler, table.

Problem 25. Clockwork toy. Determine the energy stored by the spring of a clockwork toy (car) with a fixed “winding” (number of turns of the key).
Equipment: a clockwork toy of known mass, a ruler, a tripod with a foot and a clutch, an inclined plane.
Note. Wind up the toy so that its run does not exceed the length of the table.

Problem 26. Determining the density of bodies. Determine the density of the load (rubber bung) and the lever (wooden lath) using the proposed equipment.
Equipment: cargo of known mass (marked cork); lever (wooden rail); cylindrical glass ( 200 - 250 ml); a thread ( 1m); wooden ruler, a vessel with water.

Problem 27. We study the movement of the ball.
Raise the ball to a certain height above the table surface. Let's release it and observe its movement. If the collisions were absolutely elastic (sometimes they say elastic), then the ball would always jump to the same height. In reality, the height of the jumps is constantly decreasing. The time interval between successive jumps also decreases, which is clearly noticeable by ear. After some time, the jumps stop and the ball remains on the table.
1 task - theoretical.
1.1. Determine the proportion of lost (energy loss factor) energy after the first, second, third bounce.
1.2. Get the dependence of time on the number of bounces.

2 task - experimental.
2.1. Direct method, using a ruler, determine the coefficient of energy loss after the first, second, third impact.
It is possible to determine the energy loss coefficient using a method based on measuring the total time of the ball's movement from the moment it is thrown from a height H to the moment the bounces stop. To do this, you have to establish the relationship between the total travel time and the energy loss coefficient.
2.2. Determine the energy loss factor using a method based on measuring the total time of the ball's movement.
3. Errors.
3.1. Compare the measurement errors of the energy loss factor in paragraphs 2.1 and 2.2.

Problem 28.

  • Find the mass of the test tube given to you and its outer and inner diameters.
  • Calculate theoretically at what minimum height h min and highest altitude h max of the water poured into the test tube, it will float steadily in a vertical position, and find the numerical values ​​using the results of the first paragraph.
  • Determine h min and h max experimentally and compare with the results of point 2.

Equipment. A test tube of unknown mass with a glued scale, a vessel with water, a glass, a sheet of graph paper, a thread.
Note. It is forbidden to peel off the scale from the test tube!

Problem 29. Angle between mirrors. Determine the dihedral angle between mirrors with the greatest accuracy.
Equipment. Two mirror system, measuring tape, 3 pins, cardboard sheet.

Problem 30. Spherical segment.
A spherical segment is a body bounded by a spherical surface and a plane. Using this equipment, build a graph of the dependence of the volume V spherical segment of unit radius r = 1 from his height h.
Note. The formula for the volume of a spherical segment is not supposed to be known. Take the density of water equal to 1.0 g/cm 3 .
Equipment. A glass of water, a tennis ball of known mass m with a puncture, a syringe with a needle, a sheet of graph paper, adhesive tape, scissors.

Problem 31. Snow with water.
Determine mass fraction snow in a mixture of snow and water at the time of issue.
Equipment. A mixture of snow and ice, a thermometer, a watch.
Note. Specific heat capacity of water с = 4200 J/(kg × °С), specific heat ice melting λ = 335 kJ/kg.

Problem 32. Adjustable "black box".
In the "black box", which has 3 outputs, an electrical circuit is assembled, consisting of several resistors with a constant resistance and one variable resistor. The resistance of the variable resistor can be changed from zero to some maximum value R o with the adjusting knob brought out.
Using an ohmmeter, examine the circuit of the "black box" and, assuming that the number of resistors in it is minimal,

  • draw a diagram of an electrical circuit enclosed in a "black box";
  • calculate the resistance of fixed resistors and the value of R o ;
  • evaluate the accuracy of the resistance values ​​you calculated.

Problem 33. Measurement of electrical resistances.
Determine the resistance of the voltmeter, battery and resistor. It is known that a real battery can be represented as an ideal one, connected in series with some resistor, and a real voltmeter - as an ideal one, in parallel with which a resistor is connected.
Equipment. Battery, voltmeter, resistor with unknown resistance, resistor with known resistance.

Problem 34. Weighing ultra-light loads.
Using the proposed equipment, determine the mass m of a piece of foil.
Equipment. A jar of water, a piece of Styrofoam, a set of nails, wooden toothpicks, a ruler with millimeter divisions or graph paper, a sharpened pencil, foil, napkins.

Problem 35.
Determine the current-voltage characteristic (CVC) of the "black box" ( CJ). Describe the method of taking the CVC and build its graph. Estimate the errors.
Equipment. CJ, limiting resistor with a known resistance R, multimeter in voltmeter mode, adjustable current source, connecting wires, graph paper.
Attention. connect CJ to the current source bypassing the limiting resistor is strictly prohibited.

Problem 36. Soft spring.

  • Experimentally investigate the dependence of the elongation of a soft spring under the action of its own weight on the number of coils of the spring. Give a theoretical explanation of the relationship found.
  • Determine the coefficient of elasticity and the mass of the spring.
  • Investigate the dependence of the period of oscillation of the spring on its number of turns.

Equipment: a soft spring, a tripod with a foot, a tape measure, a clock with a second hand, a ball of plasticine with a mass m = 10 g, graph paper.

Problem 37. Wire Density.
Determine the density of the wire. Breaking the wire is not allowed.
Equipment: piece of wire, graph paper, thread, water, vessel.
Note. Density of water 1000 kg/m3.

Problem 38. Friction coefficient.
Determine the sliding friction coefficient of the bobbin material on wood. The axis of the bobbin must be horizontal.
Equipment: bobbin, thread length 0.5 m, wooden ruler fixed at an angle in a tripod, graph paper.
Note. During the work it is forbidden to change the position of the ruler.

Problem 39. Share of mechanical energy.
Determine the fraction of mechanical energy lost by the ball when falling without initial velocity from a height 1m.
Equipment: tennis ball, ruler length 1.5 m, sheet of white paper format A4, sheet of carbon paper, glass plate, ruler; brick.
Note: for small deformations of the ball, one can (but not necessarily) consider Hooke's law to be valid.

Problem 40. A vessel with water "black box".
The "black box" is a vessel with water, into which a thread is lowered, on which two weights are fixed at some distance from each other. Find the masses of the loads and their densities. Estimate the size of the loads, the distance between them and the level of water in the vessel.
Equipment: "black box", dynamometer, graph paper.

Problem 41. Optical "black box".
An optical "black box" consists of two lenses, one of which is converging and the other is diverging. Determine their focal lengths.
Equipment: a tube with two lenses (optical "black" box), a light bulb, a current source, a ruler, a screen with a sheet of graph paper, a sheet of graph paper.
Note. The use of light from a distant source is allowed. It is not allowed to bring the light bulb close to the lenses (that is, closer than the racks allow).